Đến nội dung

tuan101293 nội dung

Có 316 mục bởi tuan101293 (Tìm giới hạn từ 29-04-2020)



Sắp theo                Sắp xếp  

#221024 Mệnh đề tương đương

Đã gửi bởi tuan101293 on 20-11-2009 - 21:26 trong Đại số

Cho hàm số y=f(x)= ax+b
Biết f(-1)<f(-2) ; f(1)>f(2) và f(1999)=2000 . Tính : f(2010)

Bài này ko làm đc đâu em ơi.
hai cái đầu suy ra a<0.
với mỗi a<0 ta luôn chọn đc 1 số b:f(1999)=2000
khi đó f(2010) thay đổi.



#221025 Mệnh đề tương đương

Đã gửi bởi tuan101293 on 20-11-2009 - 21:39 trong Đại số

+,nếu tồn tại i khác j mà f(j)=f(i) suy ra i=-f(f(m+1))-f(m+f(f(i)))=-f(f(m+1))-f(m+f(f(j)))=j suy ra vô lý
suy ra f đơn ánh
+,cho m=0 suy ra f(f(f(n)))=-f(f(1))-n
suy ra f là toàn ánh
suy ra f là song ánh



#454598 Tính giới hạn.

Đã gửi bởi tuan101293 on 02-10-2013 - 01:44 trong Giải tích

trong mục toán đại học thì bạn cứ khai triển taylor quanh lân cận 0 là ra hết mà (xài cái o(x) hay O(x)) đó 

ví dụ bài 1:

$e^x-e^{-x} = (1+x+\frac{x^2}{2}+\frac{x^3}{6}) - (1-x+\frac{x^2}{2}-\frac{x^3}{6})+o(x^3) = 2x+\frac{x^3}{3}+o(x^3)$

$sin(x) = x-\frac{x^3}{6}+o(x^3)$

nên $lim = 2$




#251909 Bất đẳng thức và cực trị (THCS)

Đã gửi bởi tuan101293 on 22-01-2011 - 19:51 trong Bất đẳng thức và cực trị

Có ai giúp em với !
"bdt Finsler hadwiger " là gì thế??
em chưa học cái này

Đây nè em
http://planetmath.or...Inequality.html



#251914 Bất đẳng thức và cực trị (THCS)

Đã gửi bởi tuan101293 on 22-01-2011 - 19:59 trong Bất đẳng thức và cực trị

giúp mình!
Cho a, b, c > 0 thỏa mãn a+b+c = 3.Chứng minh rằng:
$12(\dfrac{1}{a} + \dfrac{1}{b} + \dfrac{1}{c}) \geq 4(a^3 + b^3 + c^3) + 21$
(hơi khó đấy! )

Dấu = ở đâu nhỉ??
sai đề chăng?



#253110 Bất đẳng thức và cực trị (THCS)

Đã gửi bởi tuan101293 on 11-02-2011 - 21:07 trong Bất đẳng thức và cực trị

Trong đề thi HSG Trung Quốc
Cho tam giác ABC nhọn, chứng minh
$\dfrac{cos^2 A}{cos A +1}$ + $\dfrac{cos^2 B}{cos B +1}$ + $\dfrac{cos^2 C}{cos C +1}$ $\geq$ $\dfrac{1}{2}$

http://www.artofprob...v...=59649&ml=1



#253365 Bất đẳng thức và cực trị (THCS)

Đã gửi bởi tuan101293 on 17-02-2011 - 15:26 trong Bất đẳng thức và cực trị

bài này ko khó nè!
$Let a,b,c > 0 such that : a+b+c = 1 . Prove that :$

$\sum \dfrac{a^2}{b} \geq 3\sum a^2$

Ta có
$VT=\sum_{cyc} \dfrac{a^4}{a^2b}\ge \dfrac{(\sum a^2)^2}{\sum_{cyc} a^2b} $
tức là ta sẽ CM
$\sum a^2\ge 3\sum_{cyc} a^2b$
mà $\sum a^2=(\sum a^2)(\sum a)$
nên ta cần cm
$\sum a^3+\sum_{cyc} ab^2\ge 2\sum_{cyc} a^2b$
đúng vì $a^3+ab^2\ge 2a^2b$ ,....
ĐPCM
(ký hiệu $\sum_{cyc} a^2b=a^2b+b^2c+c^2a$)



#251865 Bất đẳng thức và cực trị (THCS)

Đã gửi bởi tuan101293 on 21-01-2011 - 21:44 trong Bất đẳng thức và cực trị

bài nữa cũng khó!
Cho a,b,c là 3 cạnh của 1 tam giác S là diện tích tam giác
CM:
$\dfrac{ab\sqrt{ab}}{a+b}+\dfrac{bc\sqrt{bc}}{b+c}+\dfrac{ca\sqrt{ca}}{c+a}\geq 2\sqrt{3}S$

Xài bdt Finsler hadwiger ta có
$4\sqrt{3}S\le 2\sum ab-\sum a^2$
ta sẽ CM
$2\sum ab-\sum a^2\le \sum \dfrac{2ab\sqrt{ab}}{a+b}$
tương đương (biến đổi S.O.S)
$\sum (\sqrt{a}-\sqrt{b})^2*[\dfrac{(a+b)(\sqrt{a}+\sqrt{b})^2-2ab}{2(a+b)}]\ge 0$
(đúng)
ĐPCM
còn bài Trung quốc phía sau thì chỉ cần fang công thức cos theo 3 cạnh,svac+schur là ra ngay



#251860 Sắp đến Tết rùi

Đã gửi bởi tuan101293 on 21-01-2011 - 21:20 trong Góc giao lưu

Ước đỗ đại học ^_^



#237825 Very difficult inequality

Đã gửi bởi tuan101293 on 21-08-2010 - 22:30 trong Bất đẳng thức - Cực trị

Bạn giải ra luôn đi nhé. Thấy anh Hùng nói như trên thì mình thấy rằng bạn nên giải ra luôn cho mọi người tham khảo.

Bạn có thể tham khảo file này

File gửi kèm




#268286 Cùng nhau thảo luận về IMO 2011

Đã gửi bởi tuan101293 on 12-07-2011 - 21:21 trong Thi HSG Quốc gia và Quốc tế

Nghe từ cục cưng mà cứ thấy làm sao ý
ko bik ý tốt hay là ý j đây



#273372 Number theory Marathon

Đã gửi bởi tuan101293 on 21-08-2011 - 11:06 trong Số học

cho em giải bằng cách này được không ? cho em ý kiến nhé!!
vì x,y là nghiệm nguyên nên ta có: x=ky ( y thuộc Z , k thuộc R)
thay vào PT ta có:
$ 2k^4y^4+1=y^2 \Leftrightarrow y^2(1-2k^4y^2)=1 \Leftrightarrow y= \dfrac{1}{ \sqrt{1-2k^4y^2} } $
vì y thuộc Z nên $ \dfrac{1}{ \sqrt{1-2k^4y^2} } $ cũng thuộc Z
từ đó ta sẽ có $ \sqrt{1-2k^4y^2} =1 \Leftrightarrow k=0 hay y=0 $

chú giải sai chỗ này : vì y thuộc Z nên cái căn kia có dạng 1/n chứ ko fai là 1
***
bài này thì xài xuống thang thôi
China TST 1993
http://www.artofprob...dc926aa#p269097



#275278 Number theory Marathon

Đã gửi bởi tuan101293 on 05-09-2011 - 12:01 trong Số học

anh hiểu nhầm ý em rùi vì y thuộc Z nên $ \sqrt{1-4k^4y^2}$ là ước của 1 (ước của 1 gồm 1 và -1(loại)) từ đó em mới giải đây là cách giải của em nên em cũng sợ sai lắm (bài này trích ở đâu zậy mấy anh để em xem đáp án)

ô hay,
$\sqrt{1-4k^4y^2}$ có nguyên đâu, chỉ hữu tỉ thôi nên nó fai có dạng 1/n



#238686 Tuyển tập chuyện cười toán học

Đã gửi bởi tuan101293 on 30-08-2010 - 12:13 trong Toán học lý thú

Mặc dù tên này có thể lấy kính vì tiền,nhưng nghe cũng hay đó.thks



#242311 gjup mh bai nay vs

Đã gửi bởi tuan101293 on 29-09-2010 - 16:54 trong Phương trình, hệ phương trình và bất phương trình

Cho: a+b+c=1
a^2+b^2+c^2=1
a^3+b^3+c^3=1
tính giá trị của P=a^2002+b^2003+c^2004
Các bạn giúp mh vs nha! bài tập ôn luyên HSG đấy! nhưng đối vs các bạn thì chăks là k khó đâu

suy ra $0=(a+b+c)^2-a^2-b^2-c^2=2(ab+bc+ca)$
Bạn chú ý đẳng thức
$1-3abc=a^3+b^3+c^3-3abc=(a^2+b^2+c^2-ab-bc-ca)(a+b+c)=1$
suy ra 3abc=0
Giả sử a=0
suy ra $b+c=1$, $b^2+c^2=1$
suy ra $0=(b+c)^2-b^2-c^2=2bc$
nên b=0,c=1 (chú ý hoán vị)
suy ra P=1



#242361 Đề thi học sinh giỏi toán 12 thành phố hà nội năm 2009-2010

Đã gửi bởi tuan101293 on 29-09-2010 - 20:39 trong Thi HSG cấp Tỉnh, Thành phố. Olympic 30-4. Đề thi và kiểm tra đội tuyển các cấp.

Bài này thì mình gọi đường thẳng đó là
y=(ax+b) (d)
đường thẳng d tiếp xúc đồ thị tại 2 điểm phân biệt khi và chỉ khi pt
$ax+b=(x^3+x^2+1)(x-1)$ có 2 nghiệm kép $x_1,x_2$
đến đây cậu giải 4 pt 4 ẩn là ra



#220425 Đề thi học sinh giỏi toán 12 thành phố hà nội năm 2009-2010

Đã gửi bởi tuan101293 on 15-11-2009 - 16:37 trong Thi HSG cấp Tỉnh, Thành phố. Olympic 30-4. Đề thi và kiểm tra đội tuyển các cấp.

Đề thi học sinh giỏi thành phố hà nội - lớp 12
Năm 2009-2010
BÀI 1
cho hàm số $y=({{x}^{2}-1})^{2}-({m+1})^{2}({1-m})^{2}$
( m là tham số)
1.Biện luận theo m số giao điểm của đồ thị hàm số với trục hoành
2.Xác định m để đồ thị hàm số cắt trục hoành tại 4 điểm phân biệt có hoành độ tương ứng lập thành cấp số công.
BÀI 2
1.giải phương trình $9\left(\sqrt{4x+1}-\sqrt{3x-2} \right)=x+3$
2.Cho dãy số $u_{n}=\dfrac{P_{n}}{{A_{n+2}}^n}$trong đó số chỉnh hợp chập n của (n+2) phần tử là ${A_{n+2}}^n}$ và $P_{n}$ là số hoán vị của tập hợp gồm n phần tử với n là số nguyên dương. Tìm $lim S_{n}= \sum\limits_{i=1}^{n} u_{i} $
BÀI 3
Cho hình lập phương ABCDA'B'C'D' cạnh bằng a .Với M là một điểm thuộc cạnh AB, Chọn điểm N thuộc cạnh D'C' sao cho AM+D'N=a
1.Chứng minh MN đi qua một điểm cố định khi M thay đổi
2.tính thể tích chóp B'.A'MCN theo a. Xác định vị trí của M để khoảng cách từ B' tới mp(A'MCN) max.Tính khoảng cách lớn nhất đó theo a.
3.Tìm quỹ tích hình chiếu vuông góc của C xuống MN khi M chạy trên AB.
BÀI 4
1. Cho hai số thực $x,y$ thỏa mãn $1\geq x\geq y>0$
chứng minh: $\dfrac{{x}^{3}{y}^{2}+{y}^{3}+{x}^{2}}{{x}^{2}+{y}^{2}+1}\geq xy$
2. Viết phương trình của đường thẳng tiếp xúc với đồ thị hàm số $y=(x-1)({x}^{3}+{x}^{2}+1)$ tại hai điểm phân biệt thuộc đồ thị hàm sô.
HẾT
*****************



#307939 $$f(f(m)+f(n))=m+n,\forall m,n \in \mathbb{N^*}...

Đã gửi bởi tuan101293 on 03-04-2012 - 15:04 trong Phương trình hàm

+,Dễ thấy f đơn ánh và với mọi $n\ge 2$ thì tồn tại s để $f(s)=n$
+,thay (m+1,n-1) vào ta có :$f(m+1)-f(m)=f(n)-f(n-1)$ nên suy ra $f(m+1)-f(m)=f(2)-f(1)$
dễ thấy f(2)>f(1) nếu ko f(n)<f(1) (vô lý)
suy ra f(m+1)>f(m). xét tiếp số s mà f(s)=2. ta thấy nếu $s\ge 3$ suy ra f(2)<2 hay f(1)<1 loại nên ta có 2 TH
+,f(2)=2 suy ra f(n)=n
+,f(1)=2 suy ra thay (1,1) vào đề bài ta có f(2f(1))=2=f(1) hay f(1)=1/2, loại
suy ra f(n)=n



#238995 Chứng minh bđt sau

Đã gửi bởi tuan101293 on 01-09-2010 - 20:36 trong Bất đẳng thức - Cực trị

Bài 1. Cho a,b,c là các số thực dương và abc=1.
Chứng minh :
$\dfrac{a}{a+{b}^{2}+{c}^{3}}+\dfrac{b}{b+{c}^{2}+{a}^{3}}+\dfrac{c}{c+{a}^{2}+{b}^{3}} \leq 1$
$ <=> \sum_{cyclic}[a(b+c^2+a^3)(c+a^2+b^3)] \leq (a+b^2+c^3)(b+c^2+a^3)(c+a^2+b^3) $
$ <=> \sum_{cyclic}a^{5}c^{3}+\sum_{cyclic}a^{5}b^{2}-\sum_{cyclic}a^{3}b-\sum_{cyclic}a^{4}c\geq 0 $
Không khó để chứng minh bởi AM-GM.

Ẹc,anh cũng rất quan tâm đến bài toán này,đã thử giải= cách quy đồng lâu rồi,được 1 nửa thì nửa còn lại sai,
@Nguyễn Thái Vũ,NOVAE,.....
NẾU MỌI NGƯờI ĐỌC KỸ THÌ ARQADY NÓI LÀ TÔI VẪN CHƯA GIẢI ĐƯỢC.VẬY MÀ MÌNH THẤY CÁC BẠN NÓI NHƯ LÀ HIỂU LỜI GIẢI CỦA ARQADY VÀ BÀO NÓ ĐÚNG.MÌNH CHỊU.............
@messi:Em giải được =amgm thì post lên cho anh được thưởng thức nhé :).thks em.



#237916 Chứng minh bđt sau

Đã gửi bởi tuan101293 on 22-08-2010 - 21:50 trong Bất đẳng thức - Cực trị

Bài 1. Cho a,b,c là các số thực dương và abc=1.
Chứng minh :
$\dfrac{a}{a+{b}^{2}+{c}^{3}}+\dfrac{b}{b+{c}^{2}+{a}^{3}}+\dfrac{c}{c+{a}^{2}+{b}^{3}} \leq 1$

Mình định dùng bunhia nhưng ko được.BDT khá đẹp nên mình hy vọng có chủ topic,hay ai đó sẽ post lời giải lên.
thks



#239013 Chứng minh bđt sau

Đã gửi bởi tuan101293 on 01-09-2010 - 21:26 trong Bất đẳng thức - Cực trị

2 BDT này sẽ đúng (có thể sai :))
$\sum a^3b^5 \ge abc \sum ab^4$
$\sum a^5b^2 \ge abc \sum a^3b$

bđt thứ 2 thì đúng,xài luôn côsi+cân bằn hệ số.
bdt thứ 1 thì sai,thử với a=8,b=1/4,c=1/2



#239030 Chứng minh bđt sau

Đã gửi bởi tuan101293 on 01-09-2010 - 22:57 trong Bất đẳng thức - Cực trị

Đúng mà anh. LHS-RHS=2048.45...

đảo 1 tí là sai mà em
a=1/2,b=1/4,c=8 thì sai



#454637 1 số bài toán vui

Đã gửi bởi tuan101293 on 02-10-2013 - 14:31 trong IQ và Toán thông minh

1, 3 người mà 2 người bất kỳ đều bị nhọ  thì cả 3 đều nhọ.

2, Nếu cô thứ 1 nói đúng suy ra cô 2 là Nhị và nói sai vào thứ 2, vô lý

    suy ra cô thứ 1 nói sai, cô thứ 1 là Nhị và ngày hôm đó là 3,5,7

   cô thứ 2 là Nhất, thứ 4 nói dối tức là cô ta cũng nói dối nên ngày hôm đó là thứ 3.

3, Cụ nói là: chỉ cần nhảy lên ngựa người kia và phi về đích là thắng mà. 




#247887 Bài toán về tính chất của dãy Fibonacci

Đã gửi bởi tuan101293 on 19-11-2010 - 16:38 trong Số học

Chứng minh rằng với mọi n nguyên dương, trong $n^2-1$ số hạng đầu của dãy Fibonacci luôn tồn tại một số hạng chia hết cho n

Ta xét các bộ sau:
$(F_0,F_1),(F_1,F_2),....,(F_{n^2-1},F_{n^2})$ theo mod n
có $n^2$ bộ,mà chỉ có $n^2-1$ bộ thứ tự theo mod n (chú ý ko có bộ (0,0))
Nên tồn tại
$(F_a,F_{a+1}) \equiv ({F_{a+b},F_{a+b+1}}) (mod n)$ (thấy ngay $b\le n^2-1$)
nên $F_x \equiv F_{x+b} (mod n)$ (CM quy nạp) với mọi $0\le x\le n^2-b$
Nên $F_b\equiv F_0\equiv 0 (mod n)$
ĐPCM



#454632 nghịch lý monty hall

Đã gửi bởi tuan101293 on 02-10-2013 - 14:13 trong IQ và Toán thông minh

cái này là conditional probability rõ ràng, sự kiện sau liên quan đến sự kiện trước, bạn rai_2601 nói không đúng nhé.

Cm thì có 1 bạn ở trên nói nghe 33+33=66 cũng đúng  nhưng để lập luận toán học thì người ta thường dùng Bayes theorem. có 1 dòng thôi à.

trước mình học cái này cũng hơi bất ngờ. Nhuwng hài ở chỗ là ai cũng nghĩ là 50% thành ra chơi ô cửa bí mật chả ai đổi =)).